7
$\begingroup$

For each $n=0,1,2,\ldots$, the central trinomial coefficient $T_n$ is defined as the coefficient of $x^n$ in the expansion of $(x^2+x+1)^n$. It is easy to see that $T_n=\sum_{k=0}^{\lfloor n/2\rfloor}\binom{n}{2k}\binom{2k}k$.

On December 7, 2019, I conjectured that $$\sum_{k=1}^\infty\frac{(105k-44)T_{k-1}}{k^2\binom{2k}k^23^{k-1}}=\frac{5\pi}{\sqrt3}+6\log3\tag{1}$$ and $$\sum_{k=2}^\infty\frac{(5k-2)T_{k-1}}{k^2\binom{2k}k^2(k-1)3^{k-1}}=\frac{21-2\sqrt3\,\pi-9\log3}{12}.\tag{2}$$ As the two series converge very fast, it is easy to check (1) and (2) numerically. The two identities and related congruences appear in Section 10 of my recent preprint New series for powers of $\pi$ and related congruences. I'm unable to find proofs of $(1)$ and $(2)$. So, here I ask the following question.

Question. How to prove the conjectural identities $(1)$ and $(2)$?

Your comments are welcome!

$\endgroup$
4
  • 4
    $\begingroup$ How did you find these identities? $\endgroup$
    – L. Milla
    Dec 13, 2019 at 16:06
  • 3
    $\begingroup$ The forms of the right-hand sides suggest looking at $\sum_{k = 2}^\infty T_{k - 1} \frac{105k^2 - 109k + 28}{k^2 {{2k}\choose{k}}^2 (k - 1) 3^{k - 1}}$ and $\sum_{k = 2}^\infty T_{k - 1} \frac{105k^2 - 99k + 24}{k^2 {{2k}\choose{k}}^2 (k - 1) 3^{k - 1}}$; have you looked at them? $\endgroup$
    – user44191
    Dec 13, 2019 at 17:34
  • $\begingroup$ @user44191 Note that $105k^2-99k+24=(105k-44)(k-1)+10(5k-2)$. So $(1)$ and $(2)$ imply that $$\sum_{k=2}^\infty\frac{(105k^2-99k+24)T_{k-1}}{k^2\binom{2k}k^2(k-1)3^{k-1}}=\frac{9-6\log3}{4}.$$ $\endgroup$ Dec 13, 2019 at 22:33
  • $\begingroup$ That's exactly why I chose those - they separate out the $\log 3$ and $\pi \sqrt{3}$ terms, if I haven't made an arithmetic mistake. $\endgroup$
    – user44191
    Dec 13, 2019 at 22:38

1 Answer 1

4
$\begingroup$

Not an answer, but a reduction to a definite integral.

First, Lagrange inversion implies that the generating function for $T_k$ is $${\cal T}(z):=(1-2z-3z^2)^{-\frac12}=((1+z)(1-3z))^{-\frac12},$$ and thus $T_k = [z^k]\ {\cal T}(z)$. Notice that $${\cal T}'(z) = \frac{1+3z}{(1+z)(1-3z)}{\cal T}(z).$$

Second, by the property of the beta function, we have $$\frac{1}{k\binom{2k}k} = B(k+1,k) = \int_0^1 x^k(1-x)^{k-1}\,{\rm d}x = \frac12 \int_0^1 x^{k-1}(1-x)^{k-1}\,{\rm d}x.$$

It follows that $$I_1:=\sum_{k\geq 1} \frac{T_{k-1}}{k^2\binom{2k}k^2 3^{k-1}} = \frac14 \int_0^1\int_0^1 {\cal T}\big( \frac{x(1-x)y(1-y)}{3} \big)\,{\rm d}x{\rm d}y$$ and $$I_2 := \sum_{k\geq 1} \frac{(k-1) T_{k-1}}{k^2\binom{2k}k^2 3^{k-1}} = \frac14 \int_0^1\int_0^1\, \frac{x(1-x)y(1-y)}3\, {\cal T}'\big( \frac{x(1-x)y(1-y)}{3} \big)\,{\rm d}x{\rm d}y.$$

Since $105k-44 = 105(k-1)+61$, the first sum in question equals $105 I_2 + 61 I_1$.

The second sum in question can be reduced to a definite integral similarly.

$\endgroup$

Your Answer

By clicking “Post Your Answer”, you agree to our terms of service and acknowledge that you have read and understand our privacy policy and code of conduct.

Not the answer you're looking for? Browse other questions tagged or ask your own question.